// Numbas version: exam_results_page_options {"name": "John's copy of Particle in equilibrium on an incline.", "extensions": [], "custom_part_types": [], "resources": [["question-resources/statics2.png", "/srv/numbas/media/question-resources/statics2.png"], ["question-resources/statics3.png", "/srv/numbas/media/question-resources/statics3.png"], ["question-resources/Particle_in_equilibrium_on_a_plane.png", "/srv/numbas/media/question-resources/Particle_in_equilibrium_on_a_plane.png"], ["question-resources/Particle_in_equilibrium_on_a_plane_solution_gxmuyuG.png", "/srv/numbas/media/question-resources/Particle_in_equilibrium_on_a_plane_solution_gxmuyuG.png"]], "navigation": {"allowregen": true, "showfrontpage": false, "preventleave": false, "typeendtoleave": false}, "question_groups": [{"pickingStrategy": "all-ordered", "questions": [{"functions": {}, "ungrouped_variables": ["F1", "F3", "F4", "theta1", "tantheta2", "theta2", "F2", "thetadp", "F2check", "F2sig"], "name": "John's copy of Particle in equilibrium on an incline.", "tags": [], "advice": "

We can draw our $x$ and $y$ axis as being parallel and perpendicular to the particle, as shown in the diagram below and then resolve the forces.

\n

\n

Resolving in the $x$ direction gives equation (1)

\n

\\begin{align}
F_3 + F_4 \\cos(90^{\\circ} - \\theta_1) - F_2 \\cos \\theta_2 & = 0,\\\\[0.5em]
F_2 \\cos \\theta_2 & = F_3 + F_4 \\sin \\theta_1, \\\\
&= \\var{F3} + \\var{F4} \\sin \\var{theta1}^{\\circ}. && (1) 
\\end{align}

\n

Resolving in the $y$ direction gives equation (2)

\n

\\begin{align}
F_1 - F_4 \\cos \\theta_1 + F_2 \\sin \\theta_2 & = 0,\\\\[0.5em]
F_2 \\sin \\theta_2 & = F_4 \\cos \\theta_1 - F_1, \\\\
&= \\var{F4} \\cos \\var{theta1}^{\\circ} - \\var{F1}. && (2) 
\\end{align}

\n

Dividing equation (2) by equation (1) gives (to 3 s.f.)

\n

\\begin{align}
\\frac{F_2 \\sin \\theta_2}{F_2 \\cos \\theta_2} & = \\frac{\\var{F4} \\cos \\var{theta1}^{\\circ} - \\var{F1}}{\\var{F3} + \\var{F4} \\sin \\var{theta1}^{\\circ}}, \\\\[0.5em]
\\tan \\theta_2 & = \\frac{\\var{F4} \\cos \\var{theta1}^{\\circ} - \\var{F1}}{\\var{F3} + \\var{F4} \\sin \\var{theta1}^{\\circ}}, \\\\[0.5em]
\\theta_2 & = \\arctan \\left(\\frac{\\var{F4} \\cos \\var{theta1}^{\\circ} - \\var{F1}}{\\var{F3} + \\var{F4} \\sin \\var{theta1}^{\\circ}}\\right), \\\\[0.5em]
& = \\var{thetadp}^{\\circ}.
\\end{align} 

\n

We can then substitute the value for $\\theta_2$ in either equation (1) or (2) to find $F_2$.

\n

Using equation (1), we get

\n

\\begin{align} 
F_2 \\cos \\theta_2 & = \\var{F3} + \\var{F4} \\sin \\var{theta1}^{\\circ}, \\\\[0.5em]
F_2 & = \\frac{\\var{F3} + \\var{F4} \\sin \\var{theta1}^{\\circ}}{\\cos \\var{thetadp}^{\\circ}}, \\\\[0.5em]
& = \\var{siground( (F3 + F4*sin(radians(theta1)))/(cos(radians(thetadp))),3)} \\ \\mathrm{N}.
\\end{align}

", "rulesets": {}, "parts": [{"prompt": "

By resolving forces in components find, to 3 significant figures, the unknown angle $\\theta_2$.

\n

$\\theta_2 = $ [[0]]

", "variableReplacements": [], "variableReplacementStrategy": "originalfirst", "gaps": [{"precisionType": "sigfig", "precisionMessage": "

You have not given your answer to the correct precision.

", "allowFractions": false, "variableReplacements": [], "precision": "3", "maxValue": "theta2", "variableReplacementStrategy": "originalfirst", "strictPrecision": false, "correctAnswerFraction": false, "showCorrectAnswer": true, "precisionPartialCredit": 0, "scripts": {}, "marks": 1, "minValue": "theta2", "type": "numberentry", "showPrecisionHint": false}], "showCorrectAnswer": true, "scripts": {}, "marks": 0, "type": "gapfill"}, {"prompt": "

Using your answer from part a) find the magnitude of the force $F_2$, in Newtons to 3 significant figures.

\n

$F_2 = $ [[0]]

", "variableReplacements": [], "variableReplacementStrategy": "originalfirst", "gaps": [{"precisionType": "sigfig", "precisionMessage": "

You have not given your answer to the correct precision.

", "allowFractions": false, "variableReplacements": [], "precision": "3", "maxValue": "F2+0.01", "variableReplacementStrategy": "originalfirst", "strictPrecision": false, "correctAnswerFraction": false, "showCorrectAnswer": true, "precisionPartialCredit": 0, "scripts": {}, "marks": 1, "minValue": "F2-0.01", "type": "numberentry", "showPrecisionHint": false}], "showCorrectAnswer": true, "scripts": {}, "marks": 0, "type": "gapfill"}], "extensions": [], "statement": "

The diagram below shows a particle in equilibrium under the action of four forces.

\n

\n

You are given the following information:

\n", "variable_groups": [], "variablesTest": {"maxRuns": 100, "condition": "F2>0"}, "preamble": {"css": "", "js": ""}, "variables": {"theta2": {"definition": "degrees(arctan(tantheta2))", "templateType": "anything", "group": "Ungrouped variables", "name": "theta2", "description": ""}, "F1": {"definition": "random(3..4#0.1)", "templateType": "randrange", "group": "Ungrouped variables", "name": "F1", "description": ""}, "F2": {"definition": "(F3 + F4*sin(radians(theta1)))/(cos(radians(thetadp)))", "templateType": "anything", "group": "Ungrouped variables", "name": "F2", "description": "

uses theta2 to 3dp

"}, "theta1": {"definition": "random(10..40#1)", "templateType": "randrange", "group": "Ungrouped variables", "name": "theta1", "description": ""}, "F4": {"definition": "random(5.1..8#0.1)", "templateType": "randrange", "group": "Ungrouped variables", "name": "F4", "description": ""}, "thetadp": {"definition": "siground(theta2,3)", "templateType": "anything", "group": "Ungrouped variables", "name": "thetadp", "description": "

theta2 to 3d.p. (part a solution will be used in part b - which will allow for rounding errors)

"}, "F2sig": {"definition": "siground(F2,3)", "templateType": "anything", "group": "Ungrouped variables", "name": "F2sig", "description": ""}, "F3": {"definition": "random(0.5..2.8#0.1)", "templateType": "randrange", "group": "Ungrouped variables", "name": "F3", "description": ""}, "tantheta2": {"definition": "(F4*cos(radians(theta1))-F1)/(F3+F4*cos(radians(90-theta1)))", "templateType": "anything", "group": "Ungrouped variables", "name": "tantheta2", "description": ""}, "F2check": {"definition": "(F3 + F4*sin(radians(theta1)))/(cos(radians(theta2)))", "templateType": "anything", "group": "Ungrouped variables", "name": "F2check", "description": "

uses theta2

"}}, "metadata": {"description": "

A particle is in equilibrium on an incline. Four forces are acting on it. You're given the angle of the incline and three forces. Resolve the forces to find the angle and magnitude of the other force.

", "licence": "Creative Commons Attribution 4.0 International"}, "type": "question", "showQuestionGroupNames": false, "question_groups": [{"name": "", "pickingStrategy": "all-ordered", "pickQuestions": 0, "questions": []}], "contributors": [{"name": "John Bridges", "profile_url": "https://numbas.mathcentre.ac.uk/accounts/profile/913/"}]}]}], "contributors": [{"name": "John Bridges", "profile_url": "https://numbas.mathcentre.ac.uk/accounts/profile/913/"}]}